ALL OF INVESTMENTS

अब Quizwiz के साथ अपने होमवर्क और परीक्षाओं को एस करें!

Joe bought a stock at $57 per share. The price promptly fell to $55. Joe held on to the stock until it again reached $57, and then sold it once he had eliminated his loss. If other investors do the same to establish a trading pattern, this would contradict:

the weak-form EMH

You believe that stock prices reflect all information that can be derived by examining market trading data such as history of past stock prices, trading volume, or short interest, but you do not believe stock prices reflect all publicly available and inside information. You are a proponent of the __________ form of the EMH.

weak

Monique is buying a 30-year Treasury bond that has a bid price of 99.02 and an ask price of 98.08. What is the price Monique should expect to pay if she is buying one share?

$982.50 (980 + 8/32)

Kevin is considering a mutual fund that invests only in technology companies. What is the best term for this type of fund? A) Sector Fund. B) Growth Fund. C) Equity Fund. D) Momentum Fund.

A) Sector Fund.

Eugene is single and in the 32% federal tax brackets. He is considering the purchase of municipal bond, issued in the state other than his residence, with a YTM of 7%. What is Eugene's tax equivalent yield on the bond?

10.29% (tax equivalent yield = municipal rate/ (1-tax bracket))

What is the Beta of the following portfolio?

1.1 (picture)

A stock has produced returns of 11 percent, 4 percent, -5 percent, -8 percent, and 9 percent for the past 5 years respectively. What is the geometric mean of these returns?

1.92

Dee Trader opens a brokerage account and purchases 300 shares of Internet Dreams at $50 per share. She borrows $4,000 from her broker to help pay for the purchase. The interest rate on the loan is 5%. What is the margin in Dees account when she first purchases the stock?

11, 000 (15,000 - 4,000)

You own a portfolio that is invested 32% in Stock A, 40% in Stock B, and the remainder in Stock C. The expected returns on Stock A, B, and C are 11.5%, 15% and 9% respectively. What is the expected return on the portfolio?

12.2 (picture)

The Stride Fixed Income Fund has year end market value of assets of EUR 650 million with EUR 30 million in market value of its liabilities. Stride reports a net asset value of EUR 45.25. The number of outstanding shares is closest to:

13,701,657 (market value of assets-market value of liabilities/outstanding shares)

Assume you manage a risky fund with an expected rate of return of 20% and a standard deviation of 32%. The T-bill rate is 4%. Your client chooses to invest 70% of a portfolio in your fund and 30% in T-bills. What is the percentage expected return of your client's portfolio?

15.2%

XYZ common stock has a beta of 1.50, while ABC common stock has a beta of 1.2. The expected return on the market is 15% and the risk-free rate is 4%. Based on the capital assets pricing model and making use of the information, the required return on the ABC common stock should be:

17.20 (0.04 + 1.2(0.5 - 0.04) = 17.2%)

XYZ common stock has a beta of 1.50, while ABC common stock has a beta of 1.2. The expected return on the market is 15% and the risk-free rate is 4%. Based on the capital assets pricing model and making use of the information, the required return on the XYZ common stock should be:

20.50 (0.04 + 1.5(0.15 - 0.04) = 20.50)

The Profitability Fund has NAV per share of $17.50 on January 1. On December 31 of the same year the fund's NAV was $19.47. Income distributions were $0.75 and the fund had capital gain distributions of $1.00. Without consider taxes and transactions costs, what rate of return did an investor receive on the Profitability fund last year?

21.26% (r = (19.47 - 17.50 + .75 + 1.00)/ 17.50)

Assume you manage a risky fund with an expected rate of return of 20% and a standard deviation of 32%. The T-bill rate is 4%. Your client chooses to invest 70% of a portfolio in your fund and 30% in T-bills. What is the percentage standard deviation of your clients portfolio?

22.4

Kelly decided to accept the risk and purchased a high growth stock. Her returns for the past five years are 30%, 25%, -40%, 12% and -10% respectively. What is the sample standard deviation of these returns?

28.77 (picture)

Assume you manage a risky fund with an expected rate of return on 20% and standard deviation of 32%. The T-bill rate is 4%. Your client wishes to invest in your risk fund a proportion y of total assets so that his portfolio has an expected return of 20%. What is the standard deviation of this new portfolio?

32 (20% = y*20% + (1-y)*4% y = 100% in the risky fund sigma - p =32%)

Diversified Portfolios had year-end assets of $279,000,000 and liabilities of $43,000,000. If Diversified's NAV was $40, how many shares must have been held in the fund?

5,900,000 ((279,000,000-43,000,000)/40)

Assume you manage a risky fund with an expected rate of return on 20% and standard deviation of 32%. The T-bill rate is 4%. Your client wishes to form a portfolio with a 24% standard deviation (by investing (y) in your fund and in T-bills). What is the proportion (y) in your risky fund would create such a portfolio?

75 (sigma-p = 24% = y*32% y = 24% / 32% = 75% in risky fund)

Assume you manage a risky fund with an expected rate of return on 20% and standard deviation of 32%. The T-bill rate is 4%. Your client wishes to invest in your risky fund a proportion y of total assets so that his portfolio has an expected return of 17%. What is the proportion (y) in your risky fund would create such a portfolio?

81.25 (17% = y*20% + (1-y)*4% 13% = y*16% y = 81.25)

Thunder Equity Fund invests solely in the top one-half in size of firms in the oil and gas industry. Thunder is most likely characterized as a(n): A) Sector fund. B) Income fund. C) Growth fund. D) Large cap fund.

A) Sector fund.

An increase in the number of shares outstanding by reducing the par value of the stock is?

A stock split

A Treasury bond has an ask price of 107.04 and a bid price of 107.00. What is the dollar price a buyer expects to pay? A) $1,071.25. B) $1,070.00. C) $1,071.13. D) $1,070.40.

A) $1,071.25. (4/32)

A coupon bond pays interest semi-annually and has an ask price of 107%. If the last interest payment was made 2 months ago and the coupon rate is 9%, what will the full "dirty" price of the bond be? A) $1,085. B) $1,100. C) $1,115. D) $1,070.

A) $1,085.

A convertible bond has a par value of $1,000, but its current market price is $750. The current price of the issuing company's stock is $17 and the conversion ratio is 30 shares. The bond's conversion premium is closest to: A) $240.00. B) $44.12. C) $25.00. D) $490.00.

A) $240.00.

Andre purchased shares of Latte, Co. for $48 using a margin account with a 50% initial margin rate and a 30% maintenance margin rate. At what price will Andre receive a margin call? A) $34.29 B) $24.00 C) $68.57 D) $14.40

A) $34.29 (Margin call price = debt/1-maintenance margin; 25/.7=34.29)

Elmira is single and in the 37% federal and 4% state tax brackets, and she is subject to the federal 3.8% Net Investment Income Tax. She is considering the purchase of a municipal bond, issued in her state of residence, with a YTM of 7%. What is Elmira's tax equivalent yield on the bond? A) 12.68% B) 11.11% C) 11.86% D) 7.29%

A) 12.68%

Mary George retires early and will have no income other than her portfolio income. Mary believes she will need $500,000 in before-tax income per year to cover her living expenses. Mary has $7 million in investable assets and inflation is expected to be 0.5%. Mary plans on donating $100,000 to the Atlas Foundation, $125,000 to the local hospital, and $33,000 her alma mater's endowment fund. Mary's return objective is closest to: A) 7.9%. B) 7.1%. C) 8.2%. D) 7.4%.

A) 7.9%. (Return Objective Formula) (500,000/(7,000,000-100,000-125,000-33,000)) + 0.5% )

The Stone Harbor Fund is a closed-end investment company with a portfolio currently worth $300 million. It has liabilities of $5 million and 9 million shares outstanding. If the fund sells for $30 a share, what is its premium or discount as a percent of NAV? A) 8.47% discount B) 9.26% discount C) 8.47% premium D) 9.26% premium

A) 8.47% discount

A stock market participant that buys shares of stock in a secondary market and holds shares as part of its inventory is most likely a(n) A) Broker-dealer. B) Self-regulatory organization. C) Stock exchange. D) Independent advisor.

A) Broker-dealer.

The authority function of a self-regulatory organization is most likely characterized by: A) Creation and enforcement of its own policies. B) Quick resolution of disputes. C) Establishment of clear standards of conduct. D) The effective management of conflicts of interest.

A) Creation and enforcement of its own policies.

The Board of Directors announces the amount and date of the next dividend on the ____ date, while the ____ date is the first date on which the purchaser of a stock is no longer entitled to the recently declared dividend. A) Declaration, ex-dividend. B) Ex-dividend, record. C) Declaration, record. D) Payment, record.

A) Declaration, ex-dividend.

Thirty-year Treasury bonds are not subject to which of the following risks? A) Default risk. B) Reinvestment rate risk. C) Purchasing power risk. D) Interest rate risk.

A) Default risk.

A financial institution hopes to form an equity mutual fund that invests solely in blue-chip stocks. The most likely piece of legislation that dictates the law surrounding the fund is the: A) Investment Company Act of 1940. B) Dodd Frank Act of 2010. C)Banking Act of 1933. D) Financial Services Modernization Act of 1999.

A) Investment Company Act of 1940.

Which of the following statements concerning a unit investment trust is correct? A) It has an unmanaged portfolio. B) It usually consists of common stocks. C) It is most appropriate for young people. D) It requires constant management attention.

A) It has an unmanaged portfolio.

Commercial paper is most likely issued by: A) Large, well-known companies. B) Commercial banks. C) The Federal Reserve bank. D) A securities exchange.

A) Large, well-known companies.

John Ball is a first-time homeowner who purchases a home for $200,000 using a down payment of $20,000. The monthly mortgage payment is $1,079.19. The annual property tax bill of $6,321.82 is most likely a: A) Liquidity constraint. B) Legal constraint. C) Unique circumstance constraint. D) Time constraint.

A) Liquidity constraint.

Andrea grew up in a family where money and investing conversations were commonplace. Her father bought her 18 shares of Brick, Co. stock for her 18th birthday and she has enjoyed watching the price fluctuate up and down over the past 5 years. She does not have any savings other than the Brick stock, and she has $70,000 in student loans that she must begin paying on this year. She knows that her budget will be tight, but feels that she can make it work as long as she can find a full-time job with benefits upon graduation from college. Andrea's ability to take risk is most likely: A) Low. B) Moderate. C) High. D) Moderate to high.

A) Low.

Companies generally attempt to make a profit. When this happens, they have choices as to what to do with the positive cash flow. Which of the following represents the two choices a company has for positive net income? A) Pay a cash dividend and/or reinvest cash flows back into the business. B) Declare a reverse stock split or reinvest cash flows back into the business. C) Declare a stock dividend or reinvest cash flows back into the business. D) Buy company stock or reinvest cash flows back into the business.

A) Pay a cash dividend and/or reinvest cash flows back into the business.

A ___________ index is one in which the value is determined solely based on the prices of the companies included in the index and does not take market capitalization into consideration: A) Price-weighted index. B) Cash flow index. C) Market Value index D) Equal-weighted index.

A) Price-weighted index.

A business seeking access to capital from investors will sell securities in the: A) Primary market. B)Treasury market. C) Secondary market. D) Competitive bid market.

A) Primary market.

Which of the following statements is not correct? A) Repurchase agreements typically have maturities equal to that of commercial paper. B) Yields on bankers acceptances are typically higher than commercial paper since there is additional risk. C) Commercial paper is not considered default risk free. D) Negotiable CDs are deposits of $100,000 or more and are tradable in the secondary market.

A) Repurchase agreements typically have maturities equal to that of commercial paper.

Which of the following is not a determinant of whether a financial professional must register as an investment advisor? A) The financial professional executes trades based on recommendations about securities. B) The financial professional is compensated for providing advice relating to securities. C) The financial professional is in the business of providing advice about securities. D) The financial professional provides advice about securities.

A) The financial professional executes trades based on recommendations about securities.

Which of the following is not part of the Securities and Exchange Commission's mission? A) To insure against large losses for issuers in the primary market. B) To maintain fair, orderly, and efficient markets. C) To protect investors. D) To facilitate capital formation.

A) To insure against large losses for issuers in the primary market.

Which of the following is true about bonds? A) The price an investor will pay includes accrued interest. B) Bonds can be priced based on their yield. C) All bonds are issued at a discount. D) Most bonds pay interest.

A, B, D

A coupon bond that pays interest semi-annually has a par value of $1,000, matures in 7 years and has a yield to maturity of 7.5%. If the annual coupon rate is 9%, what is the approximate value of the bond today? A. $1,081. B. $1,083. C. $856. D. $1,000.

A. $1,081.

Crazy Fans Inc pays a current dividend of $1.36 with a growth rate of 2.9%. Crazy Fans shareholders require an 11.1% rate of return. The value of the stock using the dividend discount model is closest to: A. $17.07. B. $16.58. C. $16.84. D. $17.28.

A. $17.07.

LEP Tech is expected to pay its first dividend in exactly one year of $1.25 per share. The annual growth rate in dividends is expected to be 6% and LEP's shareholders require a return of 12%. The value of the stock is closest to: A. $20.83 B. $22.08 C. $23.49 D. $23.68

A. $20.83

A firm with a price-earnings ratio of 8.39 has earnings per share of $5.38. This firm will have an expected stock price closest to: A. $45.14. B. $44.16. C. $42.93. D. $43.38.

A. $45.14.

Anderson bought a bond with a modified duration of 11.20. By approximately what percentage will the bond price change assuming interest rates increase by 90 basis points? A. -10.08%. B. -11.20%. C. +11.20%. D. +10.08%.

A. -10.08%.

Steve invested in the Hyper Growth mutual fund 5 years ago. His returns were 26%, -10%, 15%, 3% and 31%, respectively. What was his arithmetic average return over the five years? A. 13% B. 10%. C. 11%. D. 12%.

A. 13%

Linda just purchased a Louisiana general obligation bond with a yield of 3%. She is in the 24% federal bracket and 4% state bracket. If Linda lives in Louisiana, what is the equivalent yield on a corporate bond (assume she is not subject to the federal 3.8% Net Investment Income Tax)? A. 4.17%. B. 3.95%. C. 4.40%. D. 3.13%.

A. 4.17%.

A firm in the water bottling industry pays a current dividend of $2.19. Its earnings per share is $8.36 and an analysis of the financial statements shows a return on equity of 9.32%. The sustainable growth rate is closest to: A. 6.88%. B. 5.27%. C. 5.81%. D. 4.92%.

A. 6.88%.

Primary determinants of good security market include of the following except: A. A physical location B. Price discovery C. liquidity D. none of the above

A. A physical location

A characteristic of behavioral finance least likely includes an investor that A. Are risk averse. B. Fails to completely process new information. C. Has emotional biases. D. Takes shortcuts when making allocation decisions.

A. Are risk averse.

Dr. David decides to invest $3 million in short-term fixed-income securities with an average duration of 3 years and $3 million in longer-term fixed-income securities with an average duration of 7 years. What type of strategy is Dr. David using? A. Barbell strategy. B. Bullet strategy. C. Long-short strategy. D. Intermarket spread strategy.

A. Barbell strategy.

The _____ is a graphical representation of expected return and risk, as measured by standard deviation. A. CML. B. APT. C. Efficient frontier. D. SML.

A. CML.

Donna is considering purchasing a bond. She is in the 30% tax bracket. Which one should she purchase if she is only concerned about YTM? A. California 10-year bond, paying 4% semi-annually, priced at $1,131.99 B. State Street 10-year bond, paying 2% semi-annually, priced at $897.99 C. Florida general obligation 10-year bond, paying 3% semi-annually, priced at $1,071.46 D. Acme 10-year bond, paying 5% semi-annually, priced at $1,171.69

A. California 10-year bond, paying 4% semi-annually, priced at $1,131.99

Which of the following correlations represents the weakest relationship between two variables? A. Correlation = +0.18. B. Correlation = +0.88. C. Correlation = -0.40. D. Correlation = -1.00.

A. Correlation = +0.18.

Relative valuation models least likely include: A. Dividend discount model. B. Price-to-book multiple. C. Price-to-equity multiple. D. Price-to-sales multiple

A. Dividend discount model.

According to Porter's 5 competitive forces used in industry analysis, rivalry among companies in the same industry tends to increase under which of the following conditions? A. Firms in capital intensive industries characterized by high fixed costs produce high outputs when demand is low. B. A large number of substitute products are available. C. Entry barriers such as labor agreements prevent uncompetitive companies from exiting the industry. D. There are several companies of varying size, with only one or two dominant firms in the industry.

A. Firms in capital intensive industries characterized by high fixed costs produce high outputs when demand is low.

Which of the following statements concerning risk and return is not correct? A. Inflation risk, or purchasing power risk, is the variability in securities returns caused by a decline in the purchasing power of the invested dollars. B. Market risk includes a wide range of factors, including business cycles, changes in interest rates, global conflicts, as well as changes in consumer preferences. C. Total return = yield + price change. D. Exchange rate risk is the risk that currency fluctuations will cause an adverse effect on the return from an investment.

A. Inflation risk, or purchasing power risk, is the variability in securities returns caused by a decline in the purchasing power of the invested dollars.

Which of the following statements about investment risk is (are) correct? A. Liquidity risk is the risk that an investment may not be able to be bought or sold quickly without a significant price concession. B. Financial risk is associated with the use of debt as a portion of the capital structure in lieu of equity and magnifies losses but not gains. C. Both a and b. D. Neither a nor b.

A. Liquidity risk is the risk that an investment may not be able to be bought or sold quickly without a significant price concession.

Which tools can help to determine an optimal investment portfolio? A. Mean-variance optimization model B. Fundamental Analysis C. Tactical asset allocation D. All of these

A. Mean-variance optimization model

Two years ago Clyde paid $144,000 for 4,000 shares PLT stock. The market value of PLT has been fluctuating wildly and is expected to continue to do so for the next several months. Earlier this year Clyde sold another stock for a gain and he would like to sell the PLT shares in December to take a loss that can offset the those gains, and then purchase the PLT shares back sometime in January. of the following will yield the greatest amount of tax savings for Clyde this year? A. Sell at $125,000 on December 10th and buy them back on January 18th B. Sell at $135,000 on December 10th and buy them back on January 18th C. Sell at $130,000 on December 10th and buy them back on January 9th D. Sell at $125,000 on December 10th and buy them back on January 9th

A. Sell at $125,000 on December 10th and buy them back on January 18th (greater loss, and therefore a greater tax savings)

ZAGG investments owns 285,000 shares of CRC, a defensive stock. After examining the stock's cash flows, its executive leadership, and its likelihood of becoming a takeover target, a research analyst estimates the intrinsic value for this firm to be $35.00. The current market price on the NASDAQ exchange is $59.23. The analyst is most likely to recommend: A. Selling the shares that are already owned. B. Shorting the shares. C. Buying the shares due to the pending economic decline. D. Buying the shares due to its intrinsic value.

A. Selling the shares that are already owned.

The geometric mean is equivalent to: A. The IRR. B. The time weighted return. C. The dollar weighted return. D. The NPV

A. The IRR.

Deke is considering purchasing a 4-year bond that is pricing such that its YTM is 3%.Which of the following is correct if this bond has a 3.6% coupon, paid semi-annually? A. The current yield is between 3% and 3.6% B. The current yield < 3% C. The current yield = 3.3% D. The current yield > 3.6%

A. The current yield is between 3% and 3.6%

Which of the following is (are) correct regarding average returns? A. The geometric mean is equivalent to IRR. B. The arithmetic mean is the average return for a series of returns and will always be less than or equal to the geometric mean. C. Neither a nor b. D. Both a and b.

A. The geometric mean is equivalent to IRR.

Based on a normal distribution, 95% of all outcomes for investment returns should fall within: A. Two standard deviations. B. Four standard deviations. C. Three standard deviations. D. One standard deviation.

A. Two standard deviations.

Assets that lie above the SML are considered to be _______. A. Undervalued based on its beta. B. Undervalued based on its standard deviation. C. Overvalued based on its standard deviation. D. Overvalued based on its beta.

A. Undervalued based on its beta.

The strong form of the EMH states that ________ must be reflected in the current stock price. A. all information, including inside information B. all security price and volume data C. all costless information D. all publicly available information

A. all information, including inside information

According to the semistrong form of the efficient markets hypothesis, ________. A. future changes in stock prices cannot be predicted from any information that is publicly available B. arbitrage between futures and cash markets should not produce extraordinary profits C. stock prices do not rapidly adjust to new information corporate insiders should have no better investment D. performance than other investors even if allowed to trade freely

A. future changes in stock prices cannot be predicted from any information that is publicly available

The primary objective of fundamental analysis is to identify ________. A. mispriced stocks B. high P/E stocks C. well-run firms D. poorly run firms

A. mispriced stocks

The preferred stock for Iron Heights pays an annual dividend of $6.00 while the firm's preferred shareholders require an 9.6% return. The value of this stock is closest to: A. $64.50. B. $61.50. C. $62.50. D. $63.50.

C. $62.50.

Sam has a $3million fixed-income portfolio that consists of Bond A, Bond B, Bond C, and Bond D. The bonds have durations of 2, 3, 8, and 10, respectively. If Sam has 20% invested in Bond A, 30% in Bond B, and 25% invested in each of the other two bonds, what is the duration for the portfolio? Assume that the correlation between the bonds is 0.5. A. 5.75. B. 5.80. C. 6.20. D. 5.50.

B. 5.80.

Dirk is considering purchasing a 6-year bond that is selling for $1,150. What is the YTM for this bond if it has a 9% coupon, paid semi-annually? A. 5.95%. B. 5.99%. C. 5.91%. D. 5.87%.

B. 5.99%.

Which of the following are excluded from the definition of investment adviser? A) Lawyers, accountants, engineers, and teachers. B) Banks and bank holding companies. C) Broker-Dealers. D) Publishers.

All of the above

ACE has EPS of $10.00 per share and has a retention ratio of 80%. Its dividend is expected to grow at a rate of 9%. If ACE stock is trading at $54.50, then the shareholder's required return is closest to: A. 9%. B. 4%. C. 11%. D. 13%.

D. 13%.

The ________ measure of returns ignores compounding.

Arithmetic average

Holly bought a 7-year bond, with a 3% coupon paid semi annually. It was priced to yield 3% when she bought it. What is the effective duration assuming a 100-basis point change in interest rates? A. 6.0561. B. 6.2775. C. 5.7598. D. 5.9827.

B. 6.2775.

The stock prices of four companies are $20, $22, $27, and $28. What is the value of a price weighted index for these stocks? A) $27.50. B) $24.25. C) $20.00. D) $26.00.

B) $24.25. (20+22+27+28/4)

You purchased 100 shares of ABC common stock on margin at $40 per share. Assume the initial margin is 60% and the maintenance margin is 30%. You will get a margin call if the stock drops below _________. (Assume the stock pays no dividends and ignore interest on the margin loan.) A) $42.62 B) $24.62 C) $32.52 D) $22.86

B) $24.62

The Hone Starboard Fund is a closed-end investment company with a portfolio currently worth $500 million. It has liabilities of $3 million and 8 million shares outstanding. If the fund sells for $65 a share, what is its premium or discount as a percent of NAV? A) 4.63% discount B) 4.63% premium C) 4.42% premium D) 4.42% discount

B) 4.63% premium

You purchased 250 shares of common stock on margin for $25 per share. The initial margin is 60% and the stock pays no dividend. Your rate of return would be __________ if you sell the stock at $32 per share. Ignore interest on margin. A) 28.00% B) 46.67% C) 21.88% D) 43.08%

B) 46.67% (Margin = 250 * $25 * 60% = $3750 Return = [250 * ($32 - $25 )]/$3750 = 46.67%)

An investor purchased shares with a market price of $70 when the initial margin requirement was 60%. If the price increases to $92, the investor's rate of return, ignoring dividends and interest, is closest to: A) 31.67% B) 52.38%. C) 120.12%. D) 44.34%.

B) 52.38%. (70*.6=42; 90-70/42=.5238)

The Frick Endowment (FE) fund provides scholarships each year to women who deferred college to raise children and are now starting or returning to school. FE's average spending rate over the past 4 years has been 6.2%, while inflation has averaged 1.4%. The endowment's portfolio generated a 15% return last year, and the risk-free rate is 2.1%. For the upcoming year, FE's return objective is closest to: A) 16.4%. B) 7.6%. C) 12.9%. D) 6.2%.

B) 7.6%. (Return objective formula) (6.2%+1.4%)

Which of the following is correct concerning bonds? A) A putable bond may be redeemed by the issuer within the terms of the indenture agreement. B) A callable bond may be redeemed by the issuer within the terms of the indenture agreement. C) A putable bond is likely to be redeemed when interest rates have declined since issuance. D) A callable bond is likely to be redeemed when interest rates have risen since issuance.

B) A callable bond may be redeemed by the issuer within the terms of the indenture agreement.

Stocks can be purchased using a combination of both cash and loans in which of the following accounts? A) A cash account. B) A margin account. C) An IRA account. D) A sweep account.

B) A margin account.

Closed-end fund shares most commonly sell: A) At the fund's net asset value. B) At a discount to the fund's net asset value. C) At a premium to the fund's net asset value. D) At a price-to-earnings ratio between 15 and 16 times.

B) At a discount to the fund's net asset value.

Mrs. Meadows has a history of insisting that her risk tolerance level is high during periods of economic prosperity, and asking that the portfolio be invested in risky stocks, and then changing her mind during economic recessions and asking that the stocks be sold. This should be noted in which of the following constraints? A) Time constraint. B) Behavioral constraint. C) Return constraint. D) Risk constraint.

B) Behavioral constraint.

Which of the following is a disadvantage of bonds for the issuing corporation? A) Interest paid by the issuing corporation on bonds is a deductible expense for the corporation for federal income tax purposes. B) Bonds typically require payment of both periodic interest and maturity value. C) Bonds (debt) can increase the return on equity through favorable leverage. D) Bonds impact shareholder control.

B) Bonds typically require payment of both periodic interest and maturity value.

In a low interest rate environment, which of the following bonds are most likely to be called? A) Floating rate bonds. B) Coupon bonds selling at a premium. C) Coupon bonds selling at a discount. D) Zero-coupon bonds.

B) Coupon bonds selling at a premium.

Which of the following statements is false? A) Every bond has a maturity value. B) Every bond has coupon payments. C) Every bond has a stated rate of interest. D) Every bond has a maturity date.

B) Every bond has coupon payments.

The least likely goal of both the CFA® and the CFP® codes of conduct is A) Enforcement of ethical standards. B) Guarantee of superior performance by portfolio managers. C) The promotion of integrity of capital markets. D) Acting in the best interest of clients.

B) Guarantee of superior performance by portfolio managers.

Mutual fund expense ratios include all of the following except? A) Management fees. B) Investment losses. C) Operating expenses. D) 12b-1 fees.

B) Investment losses.

Which of the following is most likely characterized as a low-risk hybrid fund? A) Invests in combination of money market securities and shorter term bonds. B) Invests in investment grade bonds and large company equities with brand name product lines. C) Invests in equity securities that have low levels of systematic risk. D) Invests in bonds with low interest rate risk and low default risk.

B) Invests in investment grade bonds and large company equities with brand name product lines.

The purchase of a stock in hopes that it will appreciate over time is called a: A) Callable position. B) Long position. C) Limit position. D) Short position.

B) Long position.

The primary difference between Treasury notes and bonds is: A) Coupon rate. B) Maturity. C) Taxability. D) Default risk.

B) Maturity.

Lavar Boots is a financial adviser and interviews a new client. Lavar recommends an investment in longer-term bonds because of an expectation that interest rates will fall over the next five-year period. The recommendation is most likely a part of the: A) Feedback stage. B) Planning stage. C) Accumulation stage. D) Execution stage.

B) Planning stage

The price of a bond that a buyer would pay is equal to: A) The asked price less accrued interest. B) The asked price plus accrued interest. C) The bid price less accrued interest. D) The bid price plus accrued interest.

B) The asked price plus accrued interest.

Which of the following is most likely to be a component of an investor's ability to take risk? A) The client does not follow the market because daily fluctuations are upsetting to her. B) The client has a net worth of $1.5 million. C) The client had a bad experience investing in stocks in the past. D) The client has a preference for investing in bank CDs.

B) The client has a net worth of $1.5 million.

The yield to maturity on a bond is? A) Based on the assumption that payments received are reinvested at the coupon rate of return. B) The discount rate that will establish the present value of the payments equal to the current bond price. C) Lower than the coupon rate when the bond sells at a discount, and equal to the coupon rate when the bond sells at a premium. D) None of the above.

B) The discount rate that will establish the present value of the payments equal to the current bond price.

Acme, Inc. has net earnings of $2.1 billion this year. It has 700 million shares of common stock outstanding, and it paid 25 cents per share per quarter this year as a dividend. Which of the following is correct? A) The payout ratio equals 66.67%. B) The payout ratio equals 33.33%. C) The payout ratio equals 12.5%. D) The retention ratio cannot be determined based on this information.

B) The payout ratio equals 33.33%. (Payout ratio = dividends per share/EPS ----> 2.1 billion/700 million = 3; .24*4 = 1; 1/3=.333)

A U.S. citizen who invests in foreign securities: A) Can eliminate country risk by purchasing the stocks of diverse companies within the same country. B) Will not eliminate market risk by purchasing foreign securities. C) Cannot purchase securities issued by foreign governments. D) Will experience increased gains if the foreign currency weakens in relation to the U.S. dollar.

B) Will not eliminate market risk by purchasing foreign securities.

Three years ago, an investor bought 200 shares of IBM stock at $100 per share and 200 shares of GE stock at $30 per share. On December 15 of last year, the investor sold 100 shares of IBM at $75 per share and 100 shares of GE at $35 per share. On January 10, the investor bought 100 Shares of IBM at $77 per share and 100 shares of GE at $32 per share. What are the income tax consequences the investor must report from these transactions? A. No capital gain or loss B. A long-term capital gain of $500. C. A net long-term capital loss of $2,000. D. A long-term capital loss of $2,500.

B. A long-term capital gain of $500

The Anderson bond is 5% coupon bond with semi-annual coupon payments that matures in 10 years. If the YTM for this bond is 4%, what is the value of the bond? A. $1,081.11 B. $1,081.76 C. $1,125.03 D. $1,124.35

B. $1,081.76

Cisco Tech is a growth company that has been paying a dividend of $1.00 per share for the last ten years and has recently paid the same $1 for the current year. Cisco has just created "new tech" that should revolutionize artificial intelligence and therefore has decided to pay higher dividends two years from today. That dividend will be $3.00, followed by a $6.00 dividend the next year, and a $9.00 dividend the following year. It is expected that the following dividend payments will increase by 9% annually. What is the value of Cisco if the required rate of return equals 14%? A. $112.92 B. $128.73 C. $114.46 D. $93.90

B. $128.73

Jackson Inc. has EPS of $5.625 with a retention ratio of 60%. The annual growth rate in dividends is expected to be 6% and Jackson's shareholders require a return of 11%. The stock price is closest to: A. $45.00. B. $47.70. C. $67.50. D. $71.55.

B. $47.70.

The preferred stock for CRYSTAL pays an annual dividend of $8.42 while the firm's preferred shareholders require an 13.5% return. The value of this firm is closest to: A. $62.41. B. $62.37. C. $62.39. D. $62.35.

B. $62.37.

Edgar Corp (EC) is a growth company that has never paid a dividend. The EC board of directors has decided to pay its first dividend one year from today. The first dividend will be $2.00 per share. Because of the growth expectations for the company, it is expected that the following two dividend payments will increase by 40% each year. Beyond that, the EC dividend is expected to grow at 6% annually. What is the value of EC if the required rate of return equals 11%? A. $58.59. B. $67.71. C. $61.68. D. $61.00.

B. $67.71.

A portfolio has three stocks as follows: Stock Weight Beta Stock 1 50% 2.0 Stock 2 20% 0.7 Stock 3 30% 0.6 What is the weighted beta of the portfolio? A. 1.40. B. 1.32. C. 1.60. D. 1.00.

B. 1.32 (0.5*2 + 0.2*0.7+0.3*0.6)

Stan invested in the Great Growth mutual fund 5 years ago. His returns were 60%, -20%, 10%, 0% and 25%, respectively. What was the geometric average return over the five years? A. 11%. B. 12%. C. 10%. D. 13%.

B. 12%.

FLASH Delivery has EPS of $6.00 per share and has a payout ratio of 40%. Its dividend is expected to grow at a rate of 5.25%. If FLASH stock is trading at $22.86, then the shareholder's required return is closest to: A. 16.9%. B. 16.3%. C. 14.2%. D. 15.7%.

B. 16.3%.

Jocko was just told that the expected return for Echo stock was 20%, based on the CAPM. Assuming that the market return and the risk-free rate are 12% and 4%, respectively, what is the beta for Echo? A. 1.0. B. 2.0. C. 2.5. D. 1.5.

B. 2.0.

The Nacho Equity Fund has a beta of 1.44 and a standard deviation of 20.8%. It has returned 12.9% during the past year when the return on one-year Treasury bills has been 3.2% and the return on the market was 7.2%. The Jensen's Alpha of the Nacho Equity Fund is closest to A. 7.20% B. 3.94% C. 6.19% D. 4.66%

B. 3.94%

Reese bought Acme, Inc. for $40 per share two years ago. Today, Acme is trading at $72 per share. What is the annualized return for Reese? A. 80.00%. B. 34.16%. C. 40.00%. D. 28.75%.

B. 34.16%. [(72-40)/40 = 0.8 (1 + 0.8)^1/2 - 1= 34.16%]

Jack is considering purchasing a 6-year bond that is selling for $1,150. The bond can be called in 3 years at 104. What is the YTC for this bond if it has a 9% coupon, paid semi-annually? A. 4.79%. B. 4.82%. C. 4.63%. D. 5.99%.

B. 4.82%.

Carla is considering purchasing a 35-year bond that is selling for $500. What is the YTM for this bond if it has a 2% coupon, paid semi-annually? A. 5.09%. B. 5.06%. C. 5.00%. D. 5.03%.

B. 5.06%.

James is considering purchasing an 11-year bond that is selling for $1,250. What is the current yield for this bond if it has a 6.5% coupon, paid semi-annually? A. 4.3%. B. 5.2%. C. 3.7%. D. 6.5%.

B. 5.2%.

Portfolio A has a weighted beta coefficient of 1.5 and Portfolio B has a weighted beta coefficient of 0.9. With these assumptions, which of the following statements is correct? A. Because Portfolio A has a beta greater than 1.2, it is a better choice for most investors. B. Assuming the market were to drop by 5%, Portfolio B should drop less than Portfolio A. C. Neither portfolio is as volatile as the market. D. All of these are correct.

B. Assuming the market were to drop by 5%, Portfolio B should drop less than Portfolio A.

In Porter's 5 competitive forces used in industry analysis which of the following is NOT a barrier to entry affecting the threat of new entrants? A.Government regulation. B. Availability of substitute products. C. Economies of scale in exiting companies. D. High required investment.

B. Availability of substitute products.

After examining a tech firm's cash flows, its executive leadership, and its likelihood of becoming a takeover target, a research analyst estimates the intrinsic value for this firm to be $64.50. The current market price on the NASDAQ exchange is $61.10. The analyst is most likely to recommend: B. Buying the shares. C. Selling the shares that are already owned. D. Increasing allocation in the tech sector.

B. Buying the shares.

A rational investor would most likely: A. Use emotional cues to rebalance portfolios. B. Completely and accurately process covariance data. C. Be influenced by the frame of an investment decision. D. Take shortcuts when making asset allocation decisions.

B. Completely and accurately process covariance data.

Which of the following statements is not correct? A. Correlation ranges from -1 to +1. B. Covariance equals. Standard deviation for A times standard deviation for B divided by the correlation between A and B. C. Coefficient of determination ranges from 0 to +1. D. All of the above are correct.

B. Covariance equals. Standard deviation for A times standard deviation for B divided by the correlation between A and B.

Parker, who lives in Covington, Louisiana, purchased three bonds from a company based in Brazil that were yielding 9.75% and paid a 12% coupon, semi-annually. The company went bankrupt and Parker never received his money. What type of risk is this? A. Exchange rate risk. B. Default risk. C. Interest rate risk. D. Executive risk.

B. Default risk.

Which of the following is a systematic risk? A. Country risk. B. Exchange rate risk. C. Business risk. D. Executive risk.

B. Exchange rate risk.

Which of the following statements regarding technical and fundamental analysis is true? A. Technical analysis requires an evaluation of macroeconomic conditions. B. In fundamental analysis, future cash flows are projected by analyzing a firm's financial statements. C. Technical analysis values securities based on expected future cash flows. D. Fundamental analysis is based on supply and demand.

B. In fundamental analysis, future cash flows are projected by analyzing a firm's financial statements.

Free cash flow to equity in the current year will most likely increase when a firm: A. Increases its investment in net working capital. B. Issues bonds. C. Experiences a decline in net income. D. Has substantial capital expenditures.

B. Issues bonds.

Security Y has the following returns over five years: 3%, 6%, 0%, 6%, and 3%. What is the mean return and the standard deviation (sample) for Security Y? A. Mean of 3.0% and standard deviation of 6.4%. B. Mean of 3.6% and standard deviation of 2.5%. C. Mean of 3.0% and standard deviation of 2.5%. D. Mean of 3.6% and standard deviation of 6.4%.

B. Mean of 3.6% and standard deviation of 2.5%.

Portfolio X consists of 60% in Fund A with the remainder in Fund B. The standard deviation for Fund A and Fund B is 20% and 14%, respectively. The correlation between Fund A and B is 1.0. Portfolio Z consists of 65% in Fund C with the remainder in Fund D. The standard deviation for Fund C and Fund D is 20% and 14%, respectively. The correlation between Fund C and D is 0.6. Which portfolio is the most risky? A. Portfolio Z. B. Portfolio X. C. It cannot be determined from the question. D. They have the same level of risk.

B. Portfolio X.

Colin, who is 25 years old, invested $60,000 in an S&P 500 index fund, $30,000 in a fixed income fund, and $10,000 in a money market fund. At the end of the year, his portfolio investments had the following values: index fund: $80,000, fixed income fund $35,000, money market fund $10,000. He decides to rebalance his portfolio to match the initial allocation. What type of asset allocation is Colin engaging in? A. Core-Satellite. B. Strategic. C. Mean-Variance. D. Tactical.

B. Strategic

According to technical analysis, which of the following determines market values? A. The present value of the expected future cash flows. B. Supply and demand. C. The company's competitive position within the industry. D. The book value.

B. Supply and demand.

The Ignite bond is a 20-year zero-coupon bond. If the YTM for this bond is 6%, what is the value of the bond? Assume semiannual compounding. A. $302.15. B. $293.10. C. $306.56. D. $311.80.

C. $306.56.

Which of the following statements is correct? A. A company that is a going concern cannot be valued using discounted cash flows. B. The PE ratio is an earnings valuation model. C. Relative measures of valuation are generally superior to cash flow models. D. A company that is not a going concern can generally be valued using discounted cash flows.

B. The PE ratio is an earnings valuation model.

Donna is considering purchasing a 3-year bond that is selling for $1,000. Which of the following is correct if this bond has a 4% coupon, paid semi-annually? A. The YTM > current yield. B. The YTM equals the coupon rate and current yield. C. The current yield > YTM. D. The coupon rate > current yield.

B. The YTM equals the coupon rate and current yield.

Which of the following about the CML are true? A. The portfolio at the point of tangency includes an equal proportion of stocks and bonds. B. The capital market line is tangent to the efficient frontier. C. As an investor moves away from the point of tangency toward the risk-free rate, the percentage of long-term bonds in the portfolio will increase. D. As an investor moves away from the point of tangency toward the risk-free rate, the percentage of Treasury bills in the portfolio will decrease.

B. The capital market line is tangent to the efficient frontier.

The return from the CAPM is which of the following? A. The risk-free return B. The expected return C. The actual return D. The risk-premium

B. The expected return

A firm's price-to-earnings ratio will increase when: A. The retention ratio increases. B. The payout ratio increases. C. The growth rate in dividends decreases. D. The required return increases.

B. The payout ratio increases.

Ace Inc. has net earnings of $3.6 billion this year. It has 600 million shares of common stock outstanding and it paid 75 cents per share per quarter this year. Which of the following is correct? A. The payout ratio equals 33.33% B. The retention ratio equals 50% C. The payout ratio equals 66.67% D. The retention ratio equals 33.33%

B. The retention ratio equals 50% (payout ratio = dividend/eps)

Based on a normal distribution, 99% of all outcomes for investment returns should fall within: A. One standard deviation. B. Three standard deviations. C. Two standard deviations. D. Four standard deviations.

B. Three standard deviations.

Which of the following is not part of the Securities and Exchange Commission's mission: A. To facilitate capital formation B. To insure against large losses for issuers in the primary market C. To protect investors D. To maintain fair, orderly and efficient markets

B. To insure against large losses for issuers in the primary market

Assume that a company announces unexpectedly high earnings in a particular quarter. In an efficient market one might expect ________. A. an abnormal price increase before the announcement B. an abnormal price change immediately after the announcement C. an abnormal price decrease after the announcement D. no abnormal price change before or after the announcement

B. an abnormal price change immediately after the announcement

If you believe in the ________ form of the EMH, you believe that stock prices reflect all publicly available information but not information that is available only to insiders. A. perfect B. semistrong C. weak D. strong

B. semistrong

The semi strong-form of the efficient market hypothesis implies that ________ generate abnormal returns and ________ generate abnormal returns. A. technical analysis can; fundamental analysis cannot B. technical analysis cannot; fundamental analysis cannot C. technical analysis can; fundamental analysis can D. technical analysis cannot; fundamental analysis can

B. technical analysis cannot; fundamental analysis cannot

Joe bought a stock at $57 per share. The price promptly fell to $55. Joe held on to the stock until it again reached $57, and then he sold it once he had eliminated his loss. If other investors do the same to establish a trading pattern, this would contradict ________. A. the strong-form EMH B. the weak-form EMH C. the semi strong-form EMH D. technical analysis

B. the weak-form EMH

Diversified Portfolios had year-end assets of $279,000,000 and liabilities of $43,000,000. If Diversified's NAV was $40.00, how many shares must have been held in the fund? A) 6,975,000 B) 5,601,709 C) 5,900,000 D) 6,622,400

C) 5,900,000

A firm has $5 billion outstanding in long-term bonds. The firm currently has sufficient cash flow to make the scheduled coupon payments but would struggle to make those payments in the future if a substantial weakening of the economy occurs. The most likely rating on these bonds is:

BBB

The Kraft bond is a 17-year zero-coupon bond. If the YTM for this bond is 3%, what is the value of the bond? Assume semiannual compounding. A. $623.17. B. $605.02. C. $602.77. D. $620.86.

C. $602.77.

Jacques bought 100 shares of Acclivity at $80 per share, with an initial margin of 55%. He was charged 10% margin interest annually. Two year later he sold the stock for $105 per share. Acclivity declares and pays a dividend of $2 per share each year. What was Jacques' holding period return? A) 45% B) 76% C) 50% D) 61%

C) 50% ([(ending value - beginning value) + cash flows] - interest) / equity invested = HPR%. [($105 - $80) + $4 - ($80 x 45% x 10% x 2 years)] / ($80 x 55%) = 50%.)

Which of the following types of municipal bonds has the highest credit risk? A) A general obligation bond. B) An insured revenue bond. C) A revenue bond. D) An insured general obligation bond.

C) A revenue bond.

The Stride Fixed Income Fund has year end market value of assets of EUR 650 million with EUR 30 million in market value of its liabilities. Stride reports a net asset value of EUR 45.25. The number of outstanding shares is closest to: A) 15,027,624. B) 12,331,491. C) 13,701,657. D) 14,364,641.

C) 13,701,657.

The Profitability Fund had NAV per share of $17.50 on January 1. On December 31 of the same year the fund's NAV was $19.47. Income distributions were $0.75 and the fund had capital gain distributions of $1.00. Without considering taxes and transactions costs, what rate of return did an investor receive on the Profitability fund last year? A) 26.21% B) 19.11% C) 21.26% D) 10.12%

C) 21.26% (R = ($19.47 - 17.50 + .75 + 1.00) / $17.50 = 21.26%)

Robin, age 68, is retired and is concerned about preserving the value of her portfolio. Her secondary goal is to generate income. She has a portfolio of $500,000. Which of the following portfolio allocations would you recommend for her? A) 20% bond fund, 20% market neutral fund, 20% small-cap fund, 20% S&P 500 index fund,20% growth and income fund. B) 30% S&P 500 index fund, 30% growth fund, 30% high duration bond fund, 10% money market fund C) 40% S&P 500 index fund, 50% bond fund, 10% money market fund D) 50% balanced fund, 15% short-term bond fund, 20% international equity, 15% S&P 500 index fund

C) 40% S&P 500 index fund, 50% bond fund, 10% money market fund

Which of the following is correct concerning CDs? A) Jumbo CDs are not eligible for FDIC insurance. B) CDs and Jumbo CDs are used only by small investors since larger investors can earn a higher yield. C) All CDs issued by federally insured banks are eligible for FDIC coverage up to the coverage limit. D) Jumbo CDs are non-negotiable.

C) All CDs issued by federally insured banks are eligible for FDIC coverage up to the coverage limit.

Which of the following is not one of the advantages of owning exchange-traded funds in a well- diversified portfolio? A) Pricing is real time and is available throughout the day. B) They may be shorted; therefore, investors may speculate on a downturn. C) Arbitrage pricing mechanisms do not apply, so exchange-traded funds have more stable pricing and are less volatile. D) They offer ease of trading similar to stocks.

C) Arbitrage pricing mechanisms do not apply, so exchange-traded funds have more stable pricing and are less volatile.

Which of the following is not an investment company? A) Closed-end fund. B) ETF. C) CMO. D) UIT.

C) CMO.

Harold would like to purchase shares of a large, established company. He will most likely make his purchase: A) From an underwriter. B) In the primary market. C) In the secondary market. D) From the issuing corporation.

C) In the secondary market.

Which of the following are the 3 largest segments of the U.S. bond market? A) Municipal, corporate and Treasury B) Mortgage, corporate and Money Market C) Mortgage, corporate and Treasury D) Mortgage, Federal Agency and Treasury

C) Mortgage, corporate and Treasury

Which of the following is not a correct description of the section of the investment policy statement (IPS)? A) Executive Summary: Includes risk tolerance and performance expectations. B) Monitor: Sets predetermined guidelines for rebalancing the portfolio. C) Purpose: States the expectation that the portfolio will outperform the market. D) Statement of Objectives: Includes risk and return objectives and any constraints.

C) Purpose: States the expectation that the portfolio will outperform the market.

Which of the following is not included in money market funds? A) Money market mutual funds. B) Eurodollars. C) Real estate investment trusts. D) Repurchase agreements.

C) Real estate investment trusts.

Which of the following is not correct about TIPS? A) TIPS are issued directly from the U.S. Treasury. B) With deflation, the interest payments will decrease for TIPS. C) The interest rate for TIPS changes based on inflation. D) TIPS can be purchased on a competitive or noncompetitive basis.

C) The interest rate for TIPS changes based on inflation.

An open-end mutual fund is most likely owned by: A) The investment advisory firm. B) A family of funds mutual fund company. C) The shareholders in the fund. D) The investment company.

C) The shareholders in the fund.

The Gecko bond is a 10% coupon bond with semi-annual coupon payments that matures in 20 years. If the YTM for this bond is 4%, what is the value of the bond? A. $1,742.80. B. $1,815.42. C. $1,820.66. D. $1,893.49.

C. $1,820.66.

BCOOL pays a current dividend of $0.75 per share. The annual growth rate in dividends is expected to be 5% and BCOOL's shareholders require a return of 12.5%. The value of the stock is closest to: A. $9.50 B. $11.00 C. $10.50 D. $10.00

C. $10.50

Compute the free cash flow to equity for a firm with the following conditions (each account is reported on a per share basis): NI = $9.25, Depreciation = $1.52, Proceeds from a Bond Issue = $3.00, Total Debt Repayments = $0.45, Change in Net Working Capital = -$1.00 A. $15.22 B. $12.32 C. $14.32 D. $13.22

C. $14.32

Higgins purchased 2,000 shares of Dunlap, Inc. stock for $28 per share three years ago and paid a $50 brokerage commission on the transaction. This year he sold the shares for $37 per share and paid a $50 brokerage commission on the transaction. What is the amount of his taxable gain or loss? A. $0. B. $18,000. C. $17,900. D. $74,000.

C. $17,900

Paul has $1 million saved for retirement. He expects to retire in 15 years. His retirement fund is expected to earn a nominal rate of 9%, and the inflation rate is estimated at 3%. How much money (in millions) should Paul have when he retires, in real dollars? A. $2.5m. B. $3.6m. C. $2.3m. D. $1.8m.

C. $2.3m [Real return=(1.09/1.03)-1=5.8252% FV= $1m*(1.058252)15= $2.34m]

The Nacho Equity Fund has a beta of 1.44 and a standard deviation of 20.8%. It has returned 12.9% during the past year when the return on one-year Treasury bills has been 3.2%. The Treynor Ratio of the Nacho Equity Fund is closest to A. 0.1222 B. 0.0466 C. 0.0674 D. 0.0620

C. 0.0674

Portfolio B has a standard deviation of 12% and a correlation with the market of 0.85. If the standard deviation of the market is 15%, what is the beta for B? A. 0.54. B. 0.80. C. 0.68. D. Not enough information to determine.

C. 0.68 (0.12*0.85/0.15)

The highest annual range of returns for the S&P 500 has occurred for which of the following? A. 10-year rolling returns. B. 5-year rolling returns. C. 1-year rolling returns. D. 20-year rolling returns.

C. 1-year rolling returns.

The Nacho Equity Fund has a beta of 1.44 and a standard deviation of 20.8%. It has returned 12.9% during the past year when the return on one-year Treasury bills has been 3.2%. The Coefficient of variation of the Nacho Equity Fund is closest to A. 2.14 B. 0.0620 C. 1.61 D. 0.0674

C. 1.61 (standard deviation/return)

A coupon bond that pays interest annually of $100 has a par value of $1,000, matures in 5 years, and is selling today for $894.50. What is the yield to maturity on this bond? A. 9.00%. B. 12.00%. C. 13.00%. D. 7.00%.

C. 13.00%.

Mary invests $9,600 of her $12,000 available assets into the Metro Doughnut Company with the remainder in the Safe Bond Fund. The Metro Doughnut Company and the Safe Bond Fund are positively correlated. Changes in the Safe Bond Fund explain 4% of the returns for the Metro Doughnut Company. If the Metro Doughnut Company has a standard deviation of 20% and the Safe Bond Fund has a standard deviation of 5%, what is the standard deviation of the combined portfolio? A. 17.00%. B. 16.87%. C. 16.23%. D. 17.29%.

C. 16.23%.

XYZ common stock has a beta of 0.50, while ABC common stock has a beta of 2.0. The expected return on the market is 12% and the risk-free rate is 4%. Based on the capital asset pricing model (CAPM) and making use of the information, the required return on the XYZ common stock should be ____, and the required return on ABC common stock should be ____. A. 4%; 16%. B. 6%; 24%. C. 8%; 20%. D. 10%; 28%.

C. 8%; 20%.

Brandy is considering purchasing an 8-year bond that is selling for $700. What is the current yield for this bond if it has a 6% coupon, paid semi-annually? A. 11.92%. B. 9.32%. C. 8.57%. D. 10.17%.

C. 8.57%.

Overconfidence is most likely to be displayed by: A. An individual investor with 100% allocation to a broad equity index. B. The board of trustees of a college endowment fund comprised of liberal arts professors. C. A financial advisor with a three-year record of outperformance. D. A foundation with 100% allocation to risk-free government bonds because of low risk tolerance.

C. A financial advisor with a three-year record of outperformance.

Examples of emotional biases most likely include: A. Anchoring. B. Gambler's fallacy C. Clustering illusion. D. Mental accounting.

C. Clustering illusion.

Which of the following statements is correct about duration? A. Duration can exceed the maturity for a bond, if the bond has a call feature. B. The duration of a bond increases as the coupon rate increases. C. Effective duration can accommodate bonds with embedded options. D. Modified duration measures the change in the value of a bond equally as well when interest rates increase as when interest rates decrease.

C. Effective duration can accommodate bonds with embedded options.

An investor adds a second corporate bond to her portfolio after the first bond substantially outperformed during the previous year. The first bond has been upgraded after its recent performance from CC to CCC. The second bond is currently rated CC and the investors seeks a repeat performance based on no additional information. The investor is most likely suffering from A. Anchoring. B. Representativeness. C. Gambler's fallacy. D. Hindsight bias.

C. Gambler's fallacy.

Money managers who tend to pursue strategies that their colleagues have recently been successful with most likely show signs of: A. Illusion of control. B. Clustering illusion. C. Herd mentality. D. Anchoring.

C. Herd mentality.

Which of the following statements is (are) correct? A. If a market is weak-form efficient, it is also semistrong- and strong-form efficient. B. If a market is strong-form efficient, it is also semistrong- but not weak-form efficient. C. If a market is strong-form efficient, it is also semistrong- and weak-form efficient. D. If a market is semistrong-form efficient, it is also strong-form efficient.

C. If a market is strong-form efficient, it is also semistrong- and weak-form efficient.

John and Jim are portfolio managers. John's portfolio earned 7% last year with a standard deviation of 30%, while Jim's earned 5% with a standard deviation of 25%. The risk-free rate was 3%. Which do you think was the better manager? Assume you do not know their portfolio beta. A. Jim since he only had a standard deviation of 25%. B. John since he returned 7%. C. John since his Sharpe ratio was higher. D. Jim since his Sharpe ratio was higher.

C. John since his Sharpe ratio was higher.

Byron decides to invest $3 million in fixed-income securities by buying $300,000 worth of bonds with 10 different maturities, ranging from 1-year, 2-years, all the way up to 10-years. What type of strategy is Byron using? A. Bullet strategy B. Intermarket spread strategy C. Ladder strategy D. Barbell strategy

C. Ladder strategy

Which of the following is not portfolio income? A. Interest. B. Dividends. C. Limited partnership Income. D. Capital gains.

C. Limited partnership Income.

Which of the following statements concerning risk and return is not correct? A. Total return = yield + price change. B. Market risk includes a wide range of factors, including business cycles, changes in interest rates, global conflicts, as well as changes in consumer preferences. C. Liquidity risk is a more significant risk for securities such as Treasury bills. D. Exchange rate risk is the risk that currency fluctuations will cause an adverse effect on the return from an investment.

C. Liquidity risk is a more significant risk for securities such as Treasury bills.

Information reflected in security prices in a semi-strong form market most likely NOT to include: A. Board of director election results one week after the election. B. Stock prices during the previous week. C. Private information regarding the merger of industry leaders. D. Dividend information contained in the board of director announcement.

C. Private information regarding the merger of industry leaders.

When investors perception of risk decreases, then the SML should: A. Remain parallel to the original SML but shift downwards. B. Remain parallel to the original SML but shift upwards. C. Rotate downward (clockwise). D. Rotate upwards (counterclockwise).

C. Rotate downward (clockwise).

An investor owns a well-diversified portfolio that contains the equity securities of large U.S. corporations. The beta of the portfolio is 1.0. The most appropriate benchmark for this portfolio is: A. Dow Jones Industrial Average. B. S&P Small Cap 600. C. S&P 500 Index. D. Wilshire 5000.

C. S&P 500 Index.

Which of the following is a graphical representation of expected return and beta? A. CML. B. ALA. C. SML. D. Efficient frontier.

C. SML.

Jordan has a fairly diversified portfolio and is considering adding another security to it. Which of the following is correct? A. She should add a security that is fairly highly correlated as her portfolio is already diversified. B. She should add a security that is not correlated as it will provide the most diversification. C. She should add a security that is negatively correlated to achieve the best diversification. D. None of the above are correct.

C. She should add a security that is negatively correlated to achieve the best diversification.

Which of the following would be included in the market portfolio? A. Risk-free asset. B. T-bills. C. Stocks & bonds. D. None of these

C. Stocks & bonds.

Which of the following is not correct regarding the constant growth dividend discount model? A. The model is based on the dividend one year from the valuation period. B. The model can be rearranged to determine the payout ratio. C. The model requires that the required return be greater than or equal to the growth rate of the dividend. D. The model is highly sensitive to small differences in the required rate of return or the dividend growth rate.

C. The model requires that the required return be greater than or equal to the growth rate of the dividend.

Jackson owns a twenty-year zero-coupon bond priced at $551. If interest rates increase by 50 basis points, how much will the bond price change? A. The price will decrease more than 10%. B. The price will increase less than 5%. C. The price will decrease between 5% and 10%. D. The price will decrease less than 5%

C. The price will decrease between 5% and 10%.

Which of the following would protect an investor from exchange rate risk? A. global mutual funds B. American depository receipts C. Yankee bonds D. Foreign pay bonds

C. Yankee bonds

The Fun Fund had a return last year of -20%, with a beta of 1.5. The market lost 17% over the same period. The risk-free rate was 3%. Did the fund "beat the market", given its risk? A. Maybe - it depends on the stocks in the fund. B. No - the Jensen's alpha was negative. C. Yes - the Jensen's alpha was positive D. No - the fund lost more than the market.

C. Yes - the Jensen's alpha was positive

Proponents of the EMH typically advocate ________. A. a conservative investment strategy B. an aggressive investment strategy C. a passive investment strategy D. a liberal investment strategy

C. a passive investment strategy

The weak form of the EMH states that ________ must be reflected in the current stock price. A. all information, including inside information B. all publicly available information C. all past information, including security price and volume data D. all costless information

C. all past information, including security price and volume data

The semi strong form of the EMH states that ________ must be reflected in the current stock price. A. all security price and volume data B. all information, including inside information C. all publicly available information D. all costless information

C. all publicly available information

Market anomaly refers to ________. A. an exogenous shock to the market that is sharp but not persistent B. a price or volume event that is inconsistent with historical price or volume trends C. price behavior that differs from the behavior predicted by the efficient market hypothesis D. a trading or pricing structure that interferes with efficient buying and selling of securities

C. price behavior that differs from the behavior predicted by the efficient market hypothesis

The term random walk is used in investments to refer to ________. A. stock price changes that are random but predictable B. stock prices changes that follow the pattern of past price changes C. stock price changes that are random and unpredictable D. stock prices that respond slowly to both old and new information

C. stock price changes that are random and unpredictable

If you believe in the ________ form of the EMH, you believe that stock prices reflect all relevant information, including information that is available only to insiders. A. perfect B. semistrong C. strong D. weak

C. strong

Random price movements indicate ________. A. that prices cannot equal fundamental values B. that technical analysis to uncover trends can be quite useful C. that markets are functioning efficiently D. irrational markets

C. that markets are functioning efficiently

Evidence supporting semi strong-form market efficiency suggests that investors should ________. A. rely on technical analysis to select securities B. rely on fundamental analysis to select securities C. use a passive trading strategy such as purchasing an index fund or an ETF D. select securities by throwing darts at the financial pages of the newspaper

C. use a passive trading strategy such as purchasing an index fund or an ETF

The execution stage of the investment management process least likely includes: A) Interview with the client. B) Asset allocation decisions. C) Purchase of individual securities. D) Analysis of capital market expectations.

D) Analysis of capital market expectations.

Patch Software is expected to pay its first dividend in exactly one year of $0.25 per share. The growth rate in dividends is expected to be 4.5% and Patch's shareholders require 13.8%. The stock value is closest to: A. $3.11. B. $2.42. C. $2.84. D. $2.69.

D. $2.69.

Laramie bought a 20-year zero-coupon bond for $672.93. Using the formula for modified duration, approximately what percentage will the bond price change assuming interest rates increase by 120 basis points? A. +23.53%. B. +24.00%. C. -24.00%. D. -23.53%.

D. -23.53%.

Assume you purchased 400 shares of XYZ common stock on margin at $ 50 per share from your broker. If the initial margin is 50%, the amount you borrowed from the broker is ____ . A) $8,000 B) $20,000 C) $12,000 D) $10,000

D) $10,000

Multiple Mutual Funds had year-end assets of $457,000,000 and liabilities of $17,000,000. There were 24,000,000 shares in the fund at year-end. What was Multiple Mutual's Net Asset Value? A) $18.11 B) $19.12 C) $19.04 D) $18.33

D) $18.33

The Hound Dog Growth Mutual Fund is a highly sought after investment that owns $50 million in marketable securities, has $10 million worth of liabilities, and has 2 million outstanding shares. At what price should the fund's shares trade? a) A premium above $20.00. B) It depends on the supply and demand for the shares; however, it should be close to if not exactly $20.00. C) $25.00. D) $20.00.

D) $20.00.

You short-sell 150 shares of Rock Creek Fly Fishing Co., now selling for $40 per share. If you wish to limit your loss to $2,500 , you should place a stop-buy order at _____. A) $67.56 B) $23.33 C) $16.67 D) $56.67

D) $56.67

Which of the following best describes the clean price of a corporate bond quoted at 98.03? A) $98.09. B) $98.03. C) $980.94. D) $980.30.

D) $980.30.

The issuing corporation has the option of retiring the bond, at a predetermined price, prior to the maturity date of the bond. What is this type of bond called? A) A debenture bond. B) A convertible bond. C) A retiring bond. D) A callable bond.

D) A callable bond.

Money market securities are: A) Low volatility. B) Highly liquid. C) Readily marketable. D) All of the above.

D) All of the above.

Convertible bonds typically have which of the following characteristics? A) Are likely to pay a higher yield than comparable non-convertible bonds. B) Are more likely to be converted when a company's stock underperforms. C) Allow stockholders to convert their shares into bonds at a stated ratio. D) Allow bondholders to convert their bonds into shares at a stated ratio.

D) Allow bondholders to convert their bonds into shares at a stated ratio.

Open-end funds and closed-end funds are similar except for: A) Regulation. B) Objectives. C) Investment management. D) Capitalization.

D) Capitalization.

Which of the following is correct concerning T-bills? A) Competitive bidders are guaranteed to have their orders filled up to the auction maximum. B) Non-competitive bids determine the yield at any given auction. C) Non-competitive bidders are not guaranteed to have their orders filled up to the auction maximum. D) Competitive bids determine the yield at any given auction.

D) Competitive bids determine the yield at any given auction.

Smith Funds are a group of equity mutual funds that include Smith Growth, Smith Emerging Market, and Smith Balanced. The group of funds is most likely referred to as: A) Unit investment trusts. B) Exchange-traded funds. C) Real estate investment trusts. D) Family of funds.

D) Family of funds.

Initial public offerings (IPO) allow investors the opportunity to buy shares of a company that has never been traded by the public, which is referred to as the ________________. A) Initial market. B) Opportunity market. C) Secondary market. D) Primary market

D) Primary market

The Nacho Equity Fund has a beta of 1.44 and a standard deviation of 20.8%. It has returned 12.9% during the past year when the return on one-year Treasury bills has been 3.2%. The Sharpe Ratio of the Nacho Equity Fund is closest to A. 8.958 B. 0.620 C. 6.736 D. 0.466

D. 0.466

All of the following statements regarding major securities laws are correct EXCEPT: A) The Investment Company Act of 1940 forms the backbone of financial regulation for mutual funds and hedge funds. B) The Financial Services Modernization Act (Gramm-Leach-Bliley Act) removed barriers among securities firms, financial institutions, and insurance companies. C) The Dodd Frank Act established new government agencies to reduce volatility in financial markets in response to the Great Recession. D) The Securities Act of 1933 established the Securities and Exchange Commission.

D) The Securities Act of 1933 established the Securities and Exchange Commission.

The Echo bond is a 6% coupon bond with semi-annual coupon payments that matures in 15 years. If the YTM for this bond is 4%, what is the value of the bond? A. $1,271.26. B. $1,272.92. C. $1,222.37. D. $1,223.96.

D. $1,223.96.

Three years ago, George purchased 1,000 shares of ABC, Inc. for $10 per share. He signed an agreement with the company that allowed the company to use his dividend payments to purchase additional shares for him. Over the last 5 years, George received a total of $1,200 in dividend payments, which purchased an additional 100 shares of stock. If George sells all of his shares for $24,000, what is his taxable gain? A. $24,000. B. $0. C. $14,000. D. $12,800.

D. $12,800.

Which of the following bonds are least subject to reinvestment rate risk? A. 20-year corporate bonds rated AAA paying 8% B. 20-year callable corporate bonds paying 8.5% C. 10-year Treasury notes paying 6% D. 30-year zero coupon hi-yield bonds yielding 10% E. Municipal bonds with 5 years to maturity paying 4%

D. 30-year zero coupon hi-yield bonds yielding 10%

What is the duration of a 10-year bond with a coupon rate of 6%, paid annually, and a yield to maturity of 11%? A. 6.89. B. 8.11. C. 10.00. D. 7.32.

D. 7.32

Parker is considering purchasing a 5-year bond that is selling for $1,079. Which of the following is correct if this bond has a 5% coupon, paid semi-annually? A. The coupon rate > current yield. B. The YTM < coupon rate. C. The current yield > YTM. D. All of these.

D. All of these

Rebalancing a portfolio is performed in response to changes in the economic environment, in the life cycle of the client, or to re-establish the initial asset mix. Which of the following would trigger a rebalancing situation? A. Changes in liquidity needs. B. Changes in tax circumstances. C. Changes in time horizon for goals. D. All of these

D. All of these

Standard deviation is? A. Assumes the distribution is a normal distribution. B. A statistical measure of the variation of numbers or data around the mean of those numbers or data. C. Used as a measure of risk for investors. D. All of these

D. All of these

Which of the following is not one of the assumptions of the CAPM? A. CAPM assumes no taxes or transaction costs. B. All investors are rational and have the same expectations about the relationship between risk and return for investment alternatives. C. Investors can borrow and lend at the risk-free rate of return. D. All of these are assumptions of the CAPM.

D. All of these are assumptions of the CAPM.

Drake is considering purchasing a 15-year bond that is selling for $1,123. Which of the following is correct if this bond has a 2.5% coupon, paid semi-annually? A. The current yield < coupon rate. B. The YTM < coupon rate. C. The current yield > YTM. D. All of these.

D. All of these.

Which of the following statements regarding time diversification and forecasting is NOT correct? A. The variability of an investment tends to decrease as the holding period increases. B. The further one attempts to forecast into the future, the more likely that the forecast will be wrong. C. Sensitivity analysis may be used in conjunction with forecasting to identify best-case and worst-case scenarios. D. An incorrect forecast spread across may periods will be discounted and will not significantly distort results.

D. An incorrect forecast spread across may periods will be discounted and will not significantly distort results.

Most variation in portfolio returns over an extended period of time is attributable to ______. A. Security selection. B. Market timing. C. Market cycles. D. Asset allocation.

D. Asset allocation.

Prospect theory most likely suggests investors A. Are free of emotional biases. B. Are not restricted to bounded rationality. C. Avoid making decisions heuristically. D. Avoid regret when making important financial decisions.

D. Avoid regret when making important financial decisions.

While standard deviation and beta are both measures of risk, there are differences between the two. Which of the following is correct? A. Beta measures all risk B. Beta measures only unsystematic risk C. Standard deviation and bet are different, but both measure total risk D. Beta measures only systematic risk

D. Beta measures only systematic risk

While standard deviation and beta are both measures of risk, there are differences between the two. Which of the following is correct? A. Beta measures only unsystematic risk. B. Standard deviation and beta are different, but both measure total risk. C. Beta measures all risk. D. Beta measures only systematic risk

D. Beta measures only systematic risk

Ollie wants to increase the expected return on his portfolio above that of the market portfolio. According to the CML, what should he do? A. Invest in riskier assets. B. Add stocks with higher standard deviations. C. Add stocks with higher betas. D. Borrow money at the risk-free rate of return and invest in the market portfolio.

D. Borrow money at the risk-free rate of return and invest in the market portfolio.

Which of the following statements is true about equity returns, as represented by the S&P 500 from 1928 - 2016? A. The historical average returns for the 1-year rolling return and the 20-year rolling return are significantly different (more than a 2% difference). B. While the variation in the range of returns is different, the difference is not significant between the 1-year rolling return and the 20-year rolling return. C. Both a and b are true. D. Both a and b are false.

D. Both a and b are false.

What type of risk does a portfolio have with a beta of 1.5? A. Unsystematic risk only. B. Systematic and unsystematic risk._ C. Systematic risk only. D. Cannot determine from the question.

D. Cannot determine from the question.

Which of the following statements correctly identifies when income is subject to tax? A. Recognition occurs when an asset has been sold or exchanged. B. Realization occurs when the gain on an asset is reflected on the taxpayer's return. C. As a general rule, realized gains are not recognized unless a provision in the IRC requires recognition. D. Capital gains must be realized before they can be recognized on a tax return.

D. Capital gains must be realized before they can be recognized on a tax return.

If interest rates______, then the_______of the bond will_______. A. Increase, par value, decrease. B. Increase, price, increase. C. Decrease, price, decrease. D. Decrease, price, increase.

D. Decrease, price, increase.

Which is most accurate regarding a firm's common stock dividend policy? A. Dividends are an expense appearing on the firm's income statement. B. Dividends are paid until the maturity date of the outstanding equity. C. Dividend payments represent a legal obligation to the shareholders. D. Dividend payments are discretionary and can fluctuate over time.

D. Dividend payments are discretionary and can fluctuate over time.

John and Jim are portfolio managers. John's portfolio earned 7% last year with a beta of 1.7, while Jim's earned 5% with a beta of 0.8. The risk-free rate was 3%. Which do you think was the better manager? A. John since his Treynor ratio was 0.0235 B. John since he returned 7%. C. Jim since he only had a beta of 0.8. D. Jim since his Treynor ratio was 0.025

D. Jim since his Treynor ratio was 0.025

On September 20 of Year 1, Jorge purchased 1,000 shares of LMF, Inc. common stock for $25,000. He sold the shares for $35,000 on September 20 of Year 2. Which of the following statements correctly identifies the tax consequences of this transaction? A. Jorge will not be required to recognize the gain on the transaction. B. Jorge will recognize a $10,000 ordinary gain on the sale. C. Jorge will recognize a $10,000 long-term capital gain on the sale. D. Jorge will recognize a $10,000 short-term capital gain on the sale.

D. Jorge will recognize a $10,000 short-term capital gain on the sale.

Sarah recently reviewed her portfolio as part of her year-end performance assessment. She sold several stocks that had positive results for the year but held on to two stocks that were below her purchase price. She did not re-evaluate the potential performance for these stocks. Which form of bias is she exhibiting? A. Overconfidence bias. B. Confirmation bias. C. Hindsight bias. D. Loss aversion.

D. Loss aversion.

Lisa, who lives in Georgia, is in the32% federal tax bracket and 5% state income tax bracket. Which of the following bonds that she is considering purchasing has the highest after-tax yield (assume she is not subject to the federal 3.8% Net Investment Income Tax)? A. California Municipal bond paying 2.4%. B. Treasury bond paying 3.3%. C. Corporate bond paying 3.7%. D. Louisiana Municipal bond paying 2.6%.

D. Louisiana Municipal bond paying 2.6%.

Which of the following stages of the industry life cycle is characterized by increased competition and deceleration of profitability? A. Market maturity. B. Pioneering development. C. Market decline. D. Mature growth.

D. Mature growth.

Ollie is considering two portfolios: 1) Portfolio A with a return of 12% and a standard deviation of 20% and 2) Portfolio B with a return of 6% and a standard deviation of 10%. Which of the following statements are correct about portfolio return and risk? A. If the correlation between A and B is 1.0, then the standard deviation of a 50/50 portfolio will be the same as an 80/20 portfolio. B. At a correlation of -1, 100% of the portfolio invested in B will generate the lowest portfolio standard deviation. C. Both are correct D. Neither is correct

D. Neither is correct

The "doctor" describes himself as a "swinging for the fences" type of investor. He invests in 1 stock ABC that has a beta of 0.75 and concludes that he is only taking 3/4 of the risk of the market. Is he right? A. Yes, beta is 0.75 therefore 3/4 risk of market. B. No, he has non-diversifiable risk that is greater than the market. C. Yes, while he has diversifiable risk, his risk is only 3/4 of the market. D. No, beta only measures systematic risk.

D. No, beta only measures systematic risk.

Which of the following statements is correct? A. Coefficient of variation equals average return divided by standard deviation. B. Covariance for A and B equals. Standard deviation for A times correlation between A and B divided by standard deviation for B. C. Correlation and R2 range from -1 to +1. D. None of the above are correct.

D. None of the above are correct.

Considering the CML, which of the following is correct? A. As a portfolio moves beyond the point of tangency (to the right), the allocation to equities increases. B. The point at which the CML is tangent to the efficient frontier consists of a portfolio of 50% risky assets and 50% risk-free asset. C. As a portfolio moves from the point of tangency to the risk-free rate, the percentage of bonds will increase. D. None of the above is correct.

D. None of the above is correct.

A large firm announces it has hired a design company to change its corporate logo. The new logo will be shown publicly for the first time the following day. Shareholders react by bidding up the shares by 10% over the relevant benchmark. No other information about this firm is released. The shareholder reaction can best be described as: A. Cognitive dissonance B. Mental accounting C. Anchoring D. Representativeness

D. Representativeness

When investors perception of risk increases, then the SML should: A. Remain parallel to the original SML but shift upwards. B. Rotate downward (clockwise). C. Remain parallel to the original SML but shift downwards. D. Rotate upwards (counterclockwise).

D. Rotate upwards (counterclockwise).

Keri has a fairly diversified portfolio and is considering adding another security to it. She would like the additional security to enhance the diversification of the portfolio. Which one should she add? A. Security A has a correlation of 0.95 with her portfolio. B. Security B has a correlation of 0.55 with her portfolio. C. Security C has a correlation of 0.05 with her portfolio. D. Security D has a correlation of -0.25 with her portfolio.

D. Security D has a correlation of -0.25 with her portfolio.

Which of the following would be most appropriate to use in assessing the risk adjusted investment performance of a single stock position when mostly concerned about total risk? A. Coefficient of variation B. Beta C. Treynor Ratio D. Sharpe ratio

D. Sharpe ratio

Delta, Inc. has increasing EPS of $2.00, 2.60, 3.38, and 5.71 over the last five years. During that same time, it paid dividends of $1.00, 1.10. 1.21. 1.33 and 1.46 respectively. Which of the following is correct? A. This type of company is likely a utility B. The retention ratio equals the payout ratio C. The payout ratio has been increasing over the last five years D. The retention ratio has been increasing over the last five years

D. The retention ratio has been increasing over the last five years

Brisco believes that the market premium will go up over the next two months due to expectations about the market portfolio. What should happen to the SML? A. The slope of the SML should remain the same but the SML will shift upwards. B. The slope of the SML should remain the same but the SML will shift downwards. C. The slope of the SML should decrease. D. The slope of the SML should increase.

D. The slope of the SML should increase.

Which of the following describe the definition of risk? A. A prior periodic return. B. The market premium, defined as the difference between the market and the return on a riskless asset, such as a T-bill. C. The return expected by investors over a long-term holding period. D. The uncertainty of future returns.

D. The uncertainty of future returns.

Which of the following would be most appropriate to use in assessing the risk adjusted investment performance of a well diversified portfolio? A. Coefficient of variation B. Standard deviation C. Sharpe ratio D. Treynor Ratio

D. Treynor Ratio

Proponents of the EMH think technical analysts ________. A. should focus on support levels B. should focus on resistance levels C. should focus on relative strength D. are wasting their time

D. are wasting their time

When the market risk premium rises, stock prices will ________. A. have excess volatility B. rise C. recover D. fall

D. fall

For a given stock purchase with a given amount of initial margin: A. Maintenance margin will tend to be higher for diversified accounts B. Buyers face exaggerated losses from even mild upturns in stock prices C. A maintenance margin call will be issued following a 20% decline in price D. None of the above

D. none of the above

Most evidence indicates that U.S. stock markets are ________. A. neither weak-, semi strong-, nor strong-form efficient B. strong-form efficient C. reasonably weak-form but not semi strong- or strong-form efficient D. reasonably weak-form and semi strong-form efficient

D. reasonably weak-form and semi strong-form efficient

Insiders are able to profitably trade and earn abnormal returns prior to the announcement of positive news. This is a violation of which form of efficiency? A. technical analysis B. weak-form efficiency C. semi strong-form efficiency D. strong-form efficiency

D. strong-form efficiency

Most people would readily agree that the stock market is not ________. A. efficient at all B. semi strong-form efficient C. weak-form efficient D. strong-form efficient

D. strong-form efficient

You believe that stock prices reflect all information that can be derived by examining market trading data such as the history of past stock prices, trading volume, or short interest, but you do not believe stock prices reflect all publicly available and inside information. You are a proponent of the ________ form of the EMH. A. strong B. perfect C. semistrong D. weak

D. weak

The least likely goal of both the CFA and CFP codes of conduct is:

Guarantee of superior performance by portfolio managers

Money managers who tend to pursue strategies that their colleagues have recently been successful with most likely show signs of:

Herd mentality

All of the following statements regarding U.S. Treasury securities are correct EXCEPT:

Individual investors cannot purchase treasury issued directly from the Treasury Department

A "defensive" stock would most likely have a "beta" that is:

Less than one

Sarah recently reviewed her portfolio as part of her year-end performance assessment. She sold several stocks that had positive results for the year but held on to two stocks that were below her purchase price. She did not re-evaluate the potential performance for these stocks. Which form of bias is she exhibiting.

Loss aversion

Which tools can help to determine an optimal investment portfolio?

Mean-variance optimization model

Dee Trader opens a brokerage account and purchases 300 shares of Internet Dreams at $50 per share. She borrows $4,000 from her broker to help pay for the purchase. The interest rate on the loan is 5%. If the share price falls to $30 per share by the end of the year, what is the remaining margin in her account? If the maintenance margin requirement is 30%, will she receive a margin call?

No margin call - the % margin is 53.3% ( principal*(1+interest rate) = 4,000*1.05= 4,2000; Margin on long position = equity in account /value of stock = (9,000 - 4,200)/9,000 = 0.533)

The "doctor" describes himself as a "swinging for the fences" type of investor. He invests in 1 stock ABC that has a beta of 0.75 and concludes that he is only taking 3/4 of the risk of the market. Is he right?

No, beta only measures systematic risk

Based on current dividend yields and expected capital gains, the expected rates of returns on portfolios A and B are 11% and 14% respectively. The beta of A is .8, while that of B is 1.5. The T-bill rate is currently 6%, while the expected rate of return of the S&P 500 index is 12%. The standard deviation of portfolio A is 10% annually while that of B is 31%, and that of the index is 20%. If you decided to invest in only one of these portfolios, which would you choose?

Portfolio A - its Sharpe ratio is 0.50 (Sa = .5, Sb = .26; look at alpha; when analyzing a portfolio in isolation, you would analyze using the Sharpe ratio)

A hybrid investment security that has characteristics of both common stock and bonds is called which of the following?

Preferred stock

Keri has a fairly diversified portfolio and is considering adding another security to it. She would like the additional security to enhance the diversification of the portfolio. Which one should she add?

Security D has a correlation of -0.25 with her portfolio

Jordan has a fairly diversified portfolio and is considering adding another security to it. Which of the following is correct?

She should add a security that is negatively correlated to achieve the best diversification

The geometric mean is equivalent to:

The IRR

All of the following statements regarding major securities laws are correct EXCEPT:

The Securities Act of 1933 established Securities and Exchange Commission

The price of a bond that a buyer would pay is equal to:

The ask price plus accrued interest

The holding period return on a stock is equal to ______.

The capital gain yield over the period plus the dividend yield

Which of the following is most likely to be a component of an investor's ability to take risk?

The client has a net worth of $1.5 million

An open-end mutual fund is most likely owned by:

The shareholders in the fund

A U.S. citizen who invests in foreign securities:

Will not eliminate market risk by purchasing foreign securities.

A characteristic of behavioral finance least likely includes an investor that

are risk averse

Information reflected in security prices in a semi-strong form market most likely does not include:

private information regarding the merger of industry leaders


संबंधित स्टडी सेट्स

KHP415 Final Exam Quizzes 6-12 (Fan Gao)

View Set

Texas Principles of Real Estate 1: Chapter 5 Quiz

View Set

International Business exam 4 chapter 16

View Set

Chapter 26: Truman and the Cold War (1945-1952)

View Set

Prep U Chapter 34: Assessment and Management of Patients with Inflammatory Rheumatic Disorders

View Set

Chapter 19: Preventative and corrective oral appliances

View Set

AP Gov: Freedom of Religion/Separation of Church and State

View Set